Neuro/Lung Pathophys Resource Questions
A 76-year-old man is brought to the clinic by his granddaughter for concerns of worsening memory problems. As per the granddaughter, the patient has a slow decline in his memory over the last several years. In the last year, he has become much more forgetful. He misplaces things all the time and is getting increasingly paranoid. He sometimes does not recognize his own family members. His medical history includes hypothyroidism, diabetes mellitus type 2, hypertension, and arthritis. His medications include levothyroxine, metformin, lisinopril, and aspirin. His family history includes mother and paternal grandmother with dementia. On examination, vitals are stable. The mini-mental state examination score is 14/30. His granddaughter, who is a medical student, inquires about the role of genetics and particularly apolipoprotein E (APOE) with the patient's neurological condition. You explain to her that APOE can influence the risk of acquiring this condition. Which of the APOE genotype is known to decrease the risk of this neurological condition? 1. APOE2 2.APOE3 3.APOE4 APOE1
1. APOE2 Gene called APOE can influence the risk for the late-onset type of Alzheimer's disease. Apolipoprotein E is a regulator of lipid metabolism that supports transport of lipids and injury repair in the brain. There are three types of APOE alleles- APOE2, E3 and E4. E2 allele is the rarest and appears to reduce the risk of Alzheimer's disease. APOE3 is the most common allele and does not influence the risk of AD. APOE4 allele increases the risk of Alzheimer's disease. Each APOE4 allele also lowers the age of disease onset.
A 38-year-old male with mental retardation (IQ 50), facial features that include epicanthic folds and low-set small ears, and hands pictured. here, is brought in by his elderly parents, who provide constant care. They are concerned that over the past 3 years, he has become increasingly forgetful and less interested in conversing and sharing his thoughts. If a post-mortem brain autopsy were conducted, the most likely histopathological feature in his brain underlying these changes would be: 1.Beta-amyloid plaques and neurofibrillary tangles of phosphorylated tau 2.Clumps of alpha-synuclein and ubiquitin protein in neurons 3.Multiple, scattered ischemic lesions in the cortex 4.Marked loss of cells in the head of caudate with dilated lateral ventricles 5.Many round vacuoles in the gray matter consistent with spongiform changes
1.Beta-amyloid plaques and neurofibrillary tangles of phosphorylated tau The clinical presentation is consistent with Alzheimer's disease in a patient with Down syndrome (trisomy 21). On brain autopsy, the hallmark histopathological changes of Alzheimer's disease are extracellular beta-amyloid plaques and intracellular aggregates of phosphorylated tau (neurofibrillary tangles). Patients with trisomy 21 have an increased risk of developing Alzheimer's disease and do so at a younger age than the general population (even before the age of 40). This is because the gene for amyloid precursor protein (APP) is found on chromosome 21, which is the affected chromosome in Down syndrome. APP is the precursor protein for beta-amyloid, one of the critical proteins that builds up abnormally in Alzheimer's disease. Patients with Down syndrome have a number of commonly tested features on physical exam, including a single transverse palm crease ("simian crease"). APP is cleaved sequentially by beta- and gamma-secretase, resulting in a soluble APP portion and amyloid beta-42 peptide fragments, which aggregate and form the extracellular plaques characteristic of Alzheimer's disease. Picture shows the single transverse palm crease often seen in individuals with trisomy 21. Incorrect Answers:Answer 2: These pathologic features are consistent with Lewy Bodies, the hallmark feature of Dementia with Lewy Bodies and Parkinson's Disease. The patient did not have any movement symptoms.Answer 3: These changes are suggestive of vascular dementia, caused by multiple ischemic events.Answer 4: Degeneration of the caudate with dilated lateral ventricles would be suggestive of a diagnosis of Huntington's disease; this patient did not exhibit any movement abnormalities.Answer 5: These changes are suggestive of Creutzfeldt-Jakob disease, not Alzheimer's disease. The onset of this condition would be much more rapid and is caused by prions.
A 68-year-old male is brought to the emergency department by ambulance after he was found to be altered at home. Specifically, his wife says that he fell and was unable to get back up while walking to bed. When she approached him, she found that he was unable to move his left leg. His past medical history is significant for hypertension, atrial fibrillation, and diabetes. In addition, he has a 20-pack-year smoking history. On presentation, he is found to still have difficulty moving his left leg though motor function in his left arm is completely intact. The cause of this patient's symptoms most likely occurred in an artery supplying which of the following brain regions? 1.Cingulate gyrus 2.Globus pallidus 3.Lateral medulla 4.Lingual gyrus 5.Superior temporal gyrus
1.Cingulate gyrus This patient with acute onset lower extremity weakness most likely experienced a stroke of the anterior cerebral artery, which is the artery that supplies the cingulate gyrus. The anterior cerebral artery is responsible for supplying the medial, parietal, and frontal lobes surrounding the falx cerebri. Specifically, it provides blood supply for the medial motor and somatosensory cortex surrounding the central sulcus. Since these brain regions provide motor and sensory control of the contralateral lower extremity, patients with strokes of the anterior cerebral artery will present with lower extremity deficiencies. Additional findings may include apraxia and anosmia due to lesions of other regions supplied by this artery. Incorrect Answers: Answer 2: The globus pallidus is supplied by the lenticulostriate artery; however, a lacunar infarct of this artery would present with contralateral weakness and sensory loss in both the upper and lower extremities. Answer 3: The lateral medulla is supplied by the posterior inferior cerebellar artery; however, a stroke of this artery would present with impaired sensation of the ipsilateral face and contralateral body. Answer 4: The lingual gyrus is supplied by the posterior cerebral artery; however, a stroke of this artery would present with visual defects such as hemianopsia. Answer 5: The superior temporal gyrus is supplied by the middle cerebral artery; however, a stroke of this artery would present with aphasia and upper rather than lower extremity weakness.
A 59-year-old female with a past medical history of atrial fibrillation currently on warfarin presents to the emergency department for acute onset dizziness. She was watching TV in the living room when she suddenly felt the room spin around her as she was getting up to go to the bathroom. She denies any fever, weight loss, chest pain, palpitations, shortness of breath, lightheadedness, or pain but reports difficulty walking and hiccups. A physical examination is significant for rotary nystagmus and decreased pin prick sensation throughout her left side. A magnetic resonance image (MRI) of the head is obtained and shows ischemic changes of the right lateral medulla. What other symptoms would you expect to find in this patient? 1.Decreased gag reflex 2.Language impairment 3.Left-sided tongue deviation 4.Paralysis of the right lower limb 5.Right-sided facial paralysis
1.Decreased gag reflex This patient has lateral medullary (Wallenberg) syndrome as demonstrated by her vertigo, rotary nystagmus, and decreased pain sensation in the setting of ischemic changes in the lateral medulla on MRI. Other symptoms characteristic of lateral medullary syndrome include decreased gag reflex. Lateral medullary syndrome is a result of an acute ischemic infarct of the lateral medulla oblongata located in the brainstem. It is most commonly due to occlusion of the posterior inferior cerebellar artery (PICA), a part of the posterior circulation of the brain. Patients may present with a variety of complaints due to the various neural nuclei that may be affected. Common presentation includes nausea/vomiting, vertigo, nystagmus, decreased pain and temperature sensation from the contralateral body and ipsilateral face, autonomic dysfunctions, and ataxia. It is worth noting that nucleus ambiguous effects, which include dysphagia, hoarseness, and decreased gag reflex, are specific to PICA lesions. Incorrect Answers: Answer 2: Language impairment, or aphasia, is usually present in strokes that affect the language centers of the brain (Wernicke and Broca area), which are supplied by the middle cerebral artery. Answer 3: Left-sided tongue deviation is seen in strokes in the caudal medulla which is supplied by the anterior spinal artery. Answer 4: Paralysis of the right lower limb can be seen in strokes involving the motor cortices or the internal capsule, both of which are supplied by the anterior circulation. Answer 5: Right-sided facial paralysis deviation is seen in strokes that involve the facial nucleus located at the lateral pons.
A 73-year-old male presents to his PCP with his son who reports that his father has been acting strangely. He has started staring into space throughout the day and has a limited attention span. He has been found talking to people who are not present and has gotten lost while driving twice. He has occasional urinary incontinence. His past medical history is notable for a stroke 5 years ago with residual right arm weakness, diabetes, hypertension, and hyperlipidemia. He takes aspirin, glyburide, metformin, lisinopril, hydrochlorothiazide, and atorvastatin. On examination, he is oriented to person and place but thinks the year is 1989. He is inattentive throughout the exam. He takes short steps while walking. His movements are grossly slowed. A brain biopsy in this patient would most likely reveal which of the following? 1.Eosinophilic intracytoplasmic inclusions 2.Intracellular round aggregates of hyperphosphorylated microtubule-associated protein 3.Intracellular tangles of hyperphosphorylated microtubule-associated protein 4.Large intracellular vacuoles within a spongiform cortex 5.Marked diffuse cortical atherosclerosis
1.Eosinophilic intracytoplasmic inclusions The patient in this vignette presents with impaired cognition, inattention, hallucinations, and Parkinsonian symptoms suggestive of Lewy body dementia (LBD). Eosinophilic intracytoplasmic inclusions represent clumps of alpha-synuclein protein that can be found in the brains of patients with LBD. LBD is one of the most common causes of dementia and is characterized by the triad of hallucinations, parkinsonism, and cognitive fluctuation with dementia typically appearing later in the disease process. Lewy bodies are intraneuronal, cytoplasmic, eosinophilic inclusions composed predominantly of alpha-synuclein in addition to other substances. In LBD, Lewy bodies can be found throughout the cortex in contrast to idiopathic Parkinson disease, where they are typically found in the basal ganglia and various neurotransmitter nuclei. Incorrect Answers: Answer 2: Intracellular round aggregates of hyperphosphorylated microtubule-associated protein (tau) are seen in frontotemporal dementia. These are called Pick bodies and are distinguished from neurofibrillary tau tangles seen in Alzheimer disease by the fact that they are round aggregates instead of tangles. Answer 3: Intracellular tangles of hyperphosphorylated microtubule-associated protein (tau) are seen in Alzheimer dementia. These are sometimes called neurofibrillary tangles and are distinguished from Pick bodies seen in frontotemporal dementia by the fact that they are in tangles instead of round aggregates. Answer 4: Large intracellular vacuoles within a spongiform cortex are seen in spongiform encephalopathy (Creutzfeldt-Jakob disease). In this condition, misfolded intracellular prion proteins form large intracellular vacuoles. Answer 5: Marked diffuse cortical atherosclerosis would be seen in a patient with severe vascular disease. Vascular dementia is a heterogeneous syndrome characterized by a stepwise progressive dementia due to multiple vascular lesions.
Which of the following do you expect in a patient with left middle cerebral artery obstruction? 1.Hemiparesis greatest in the arms with aphasia 2.Clumsy hand-dysarthria 3.Vertigo, nystagmus, and cranial nerve palsies 4.Hemiparesis greatest in the legs without aphasia 5.Sensory deficits only
1.Hemiparesis greatest in the arms with aphasia Left middle cerebral artery (MCA), which would result in hemiparesis on the right side (contralateral), as well as aphasia that may be expressive or receptive depending on whether the frontal or superior temporal lobe is more affected. Normally, the MCA supplies the lateral portion of the cerebral hemisphere, including frontal, parietal, and upper temporal regions. In addition to findings described above, MCA strokes may have contralateral sensation loss, hemianopsia, hyperreflexia, and Babinski sign. While left-sided strokes cause language deficits, right-sided MCA occlusions may produce visual hemianopsia with inattention to objects on the left side. Initial diagnosis of acute stroke in the emergency room requires non-contrast head CT to rule out hemorrhage; serum electrolytes, creatinine and glucose, CBC; PT/PTT; and EKG and troponins, although the last two tests should not delay administration of IV-tPA in ischemic stroke. Incorrect Answers: Answer 2: Lacunar infarcts affecting blood flow to the pons result in clumsy hand-dysarthria syndrome. Answer 3: Emboli in the vertebrobasilar circulation, from the posterior artery, that affect blood flow to the cerebellum would lead to vertigo, nystagmus, and cranial nerve palsy. Answer 4: Weakness greatest in the legs is a feature of anterior cerebral artery stroke. Answer 5: Lacunar infarcts affecting blood flow to the thalamus result in sensory-only deficits.
A 25-year-old male rugby player presents to the ER complaining of a severe headache. He is accompanied by his teammate who reports that he had a head-to-head collision with another player and briefly passed out before regaining consciousness. His past medical history is significant for a pilocytic astrocytoma as a child treated successfully with surgery. His family history is notable for stroke in his father. His temperature is 98.9°F (37.2°C), blood pressure is 160/90 mmHg, pulse is 60/min, and respirations are 20/min. On examination, he is lethargic but oriented to person, place, and time. The affected vessel in this patient directly branches from which of the following vessels? 1.Maxillary artery 2.Internal carotid artery 3.Superficial temporal artery 4.Middle cerebral artery 5.Anterior cerebral artery
1.Maxillary artery The patient in this vignette presents with a post-traumatic headache preceded by a momentary loss of consciousness, a scenario that is suggestive of epidural hematoma. Rupture of the middle meningeal artery, a branch of the maxillary artery, leads to the development of an epidural hematoma. Epidural hematomas result from head trauma that is severe enough to fracture the temporal bone and tear the middle meningeal artery, which lies deep to the temporal bone and branches from the maxillary artery. Clinically, epidural hematomas present with momentary loss of consciousness followed by a lucid period of up to two days, headache, nausea, and focal neurologic symptoms (e.g., hemiparesis). Rapid expansion of the hematoma can also lead to tentorial herniation. Incorrect Answers: Answer 2: The internal carotid arteries arise from the common carotid arteries. They are the primary arteries that supply blood to the brain along with the vertebral arteries. A bleed from the internal carotid artery will cause a devastating hemorrhagic stroke. Answer 3: The superficial temporal artery is one of the two terminal branches of the external carotid artery along with the maxillary artery. It supplies the head and neck. It is often affected by temporal arteritis and is implicated in pain associated with migraines. Answer 4: The middle cerebral artery is a branch of the internal carotid artery and supplies the lateral frontal, temporal, and parietal lobes, the globus pallidus, caudate, and putamen. A bleed from the middle cerebral artery will cause a hemorrhagic stroke that may present with contralateral face and extremity hemiparesis or hemiplegia, contralateral sensory loss, and aphasia. Answer 5: The anterior cerebral artery is a branch of the internal carotid artery and supplies the medial frontal and parietal lobes, basal ganglia, and corpus callosum. The most common site of brain aneurysms is the anterior cerebral artery. A stroke or aneurysm rupture of the anterior cerebral artery will present with contralateral hemiparesis and contralateral sensory deficits.
A 71-year-old man presents to the clinic for a routine visit. He has no active complaints. His wife states that his memory has been decreasing lately. When asked for examples of his memory loss, the wife says that he is forgetting the neighbors' names and forgot their granddaughter's birthday. What's the most likely cause of the cognitive changes in this patient? 1.Normal forgetfulness 2.Mild cognitive impairment 3.Delirium 4.Alzheimer disease
1.Normal forgetfulness Normal forgetfulness includes momentarily forgetting names and occasionally forgetting to run an errand. In mild cognitive impairment, people frequently forget people's names and begin to forget important events. Delirium changes usually occur abruptly. Also, there is waxing and waning of alertness and orientation. In Alzheimer disease, the patient may not remember knowing a person and loses the sense of time and which day it is.
A 74-year-old female is brought to the emergency department by her home health aid. The patient was eating breakfast this morning when she suddenly was unable to lift her spoon with her right hand. She attempted to get up from the table, but her right leg felt weak. One hour later in the emergency department, her strength is 0/5 in the right upper and right lower extremities. Strength is normal in her left upper and lower extremities. Sensation is normal bilaterally. An emergency CT of the head does not show signs of hemorrhage. Subsequent brain MRI shows an infarct involving the internal capsule. Which of the following is true about her disease process? 1.The most important risk factors are hypertension and diabetes 2.The most common cause is embolism originating from the left atrium 3.It is caused by ischemia to watershed areas 4.IV thrombolysis cannot be used The most important risk factors are ethnicity and sex
1.The most important risk factors are hypertension and diabetes This patient presents with a pure motor stroke, which is one of the stroke syndromes caused by lacunar infarcts. The most important predisposing risk factors for lacunar infarcts are diabetes and hypertension. Lacunar strokes are small infarcts which occur due to the occlusion of small perforating branches arising from major intracerebral arteries. Lacunar infarcts affect the deep nuclei of the brain. Depending on which perforating artery is involved, a number of clinical syndromes can arise, including the pure motor stroke described in the question stem. The pathogenesis of lacunar strokes is different than that of the classic large vessel strokes; in particular, lacunar strokes involve lipohyalinosis of the perforating vessels. Incorrect Answers: Answer 2: Atrial fibrillation can predispose to an embolism originating from the left atrium and causing stroke. However, lacunar stroke is rarely embolic in nature. Answer 3: Lacunar infarcts occur in subcortical areas, not in major vascular watershed areas of the brain. Answer 4: IV thrombolysis can be used for ischemic lacunar infarction if the patient is otherwise a candidate for thrombolysis. Answer 5: Race and sex are not known to be particularly important risk factors specifically for lacunar stroke.
A 66-year-old African American male patient is brought by his sister with gradual memory decline over the last two years. He had several incidents of wandering in the neighborhood. He can do his activities of daily living, but he has to be reminded to take his medications. He gets easily irritable, but no physical aggression is reported. His past medical history is significant for hypertension and chronic kidney disease. He denies any history of substance use. He currently takes carvedilol, amlodipine, and hydralazine. His mini-mental state examination score is 19/30. His blood pressure is 162/73 mmHg, and heart rate is 77 beats/min. Laboratory workup, including comprehensive metabolic panel, urine analysis, urine toxicology, infectious serology, thyroid panel, vitamin B12, and folate levels are unremarkable except for a serum creatinine of 1.71 mg/dl. Magnetic resonance imaging (MRI) brain and electroencephalogram (EEG) are also unremarkable. What medication can worsen the memory deficits in this patient? 1.N-methyl-D-aspartate (NMDA) receptor antagonist 2.Anticholinergic medication 3.Cholinesterase inhibitor medication 4.Monoclonal antibody against beta-amyloid
2.Anticholinergic medication Alzheimer disease (AD) is the most common type of dementia. It is characterized by an accumulation of abnormal neuritic plaques and neurofibrillary tangles. Anticholinergic drugs can cause dry mouth, urinary retention, constipation, and irritability, which can potentiate existing dementia. Anticholinergic medications should be avoided, as acetylcholine deficiency is associated with cognitive deficits in Alzheimer disease. Alzheimer disease is a gradual and progressive neurodegenerative disease caused by neuronal cell death. Environmental and behavioral approaches are beneficial, especially in managing behavioral problems. Medications that increase acetylcholine like cholinesterase inhibitors are helpful and have been approved for use in Alzheimer disease, i.e., donepezil, rivastigmine, and galantamine. Research is ongoing regarding immunologic therapies to clear amyloid plaques. NMDA receptor antagonist (memantine) is approved for moderate to severe Alzheimer dementia.
A 48-year-old male presents to the ER with a sudden-onset, severe headache. He is vomiting and appears confused. His wife, who accompanied him, says that he has not had any trauma, and that the patient has no relevant family history. He undergoes a non-contrast head CT that shows blood between the arachnoid and pia mater. What is the most likely complication from this condition? 1.Blindness 2.Arterial Vasospasm 3.Hemorrhagic shock 4.Bacterial Meningitis 5.Renal failure
2.Arterial Vasospasm The patient has had a subarachnoid hemorrhage (SAH), and arterial vasospasm is the most common complication from this disease. A subarachnoid hemorrhage is a bleed into the space between the arachnoid and pia mater that can occur spontaneously (typically from a ruptured cerebral aneurysm, such as a berry aneurysm), or from head injury. Arterial vasospasm is the most common complication following SAH and may occur in >50% of cases. It can cause delayed ischemic brain injury 2-3 days after the initial event, which can lead to permanent impairment or death. Incorrect answers: Answer 1: Though this may be a complication of cerebral damage in certain areas, this is not the most common complication. Answer 3: Because the skull is a closed compartment, it is not possible to lose enough blood in a cerebral bleed to develop shock. Answer 4&5: These are not complications of subarachnoid hemorrhage.
A 30-year-old male is brought to the emergency department following a skateboarding accident. He was jumping down a flight of stairs, lost control, and fell onto concrete without wearing a helmet. During impact, his friends say he hit his head and lost consciousness briefly before recovering. He continued to skateboard for 20 minutes but then began experiencing a severe headache and has vomited 3 times. His temperature is 98.0°F (36.7°C), blood pressure is 165/87 mmHg, pulse is 82/min, and respirations are 19/min. On exam, he is alert and oriented to person and place but not time. His pupils are 4 mm bilaterally and reactive to light. He has no focal neurological deficits. A noncontrast head CT is performed and shown. A branch from which of the following vessels is the most likely source of hemorrhage? 1.Bridging vein 2.Deep temporal artery 3.Internal carotid artery 4.Maxillary artery 5.Middle temporal artery
4.Maxillary artery This patient has a history of head trauma, loss of consciousness followed by a lucid interval, and subsequent signs of increased intracranial pressure (headache and vomiting). The noncontrast head CT demonstrates an epidural hematoma, which is likely from a middle meningeal artery (branch of the maxillary artery) hemorrhage. Epidural hematomas occur as the result of head trauma causing hemorrhage between the skull and dura mater. Classically, the patient has a lucid interval after initial loss of consciousness followed by sings of increased intracranial pressure (e.g., headache, vomiting, decreased mental status, anisocoria, and Cushing reflex). The source of hemorrhage is most often the middle meningeal artery, which is a branch of the maxillary artery. Epidural hematomas are diagnosed with noncontrast CT scans and demonstrate biconvex, hyperdense masses confined to the suture lines between the dura mater and the skull. Rapidly expanding epidural hematomas can cause mass effect leading to CN III palsy and herniation. Treatment includes urgent craniotomy and hematoma evacuation. Incorrect Answers: Answer 1: Bridging vein rupture causes a subdural hematoma which presents as increasing headaches over several days or weeks with possible changes in mental status. On nonconstrast CT, subdural hematomas appear as crescent-shaped, hyperdense lesions between the brain and skull not limited by suture lines. Answer 2: Deep temporal artery rupture results in hemorrhage outside the calvarium and would not cause an intracranial hemorrhage. The deep temporal artery branches off the maxillary artery and ascends between the temporalis and the pericranium. Answer 3: Internal carotid artery rupture can occur at the site of an aneurysm and would result in a subarachnoid hemorrhage, which would present as a sudden, severe headache (i.e., worst headache of life). On noncontrast head CT, subarachnoid hemorrhage appears as an extensive hyperdense network around the circle of Willis. Answer 5: Middle temporal artery rupture results in hemorrhage outside the calvarium and would not cause an intracranial hemorrhage. The middle temporal artery branches off the superficial temporal artery above the zygomatic arch and perforates the temporal fascia to supply the temporalis muscle.
A 5-year-old patient is brought to the ER by his parents for concerning behavior. His parents relate that over the past 3 weeks, he has had multiple episodes of staring into space, lip smacking, and clasping his hands together. The patient has his eyes open during this episode but does not respond to his parents' voice or his name. These episodes last between 1-2 minutes after which the patient appears to return back to awareness. The patient is confused after these episodes and appears not to know where he is for about 15 minutes. These episodes occur once every few days and the most recent one happened about 10 minutes before the patient arrived to the emergency department. On arrival, the patient is mildly confused and does not know where he is or what recently happened. He is slow to respond to questions and appears tired. Which of the following is the most likely diagnosis in this patient? 1.Simple partial seizure 2.Complex partial seizure 3.Generalized tonic-clonic seizure 4.Absence seizure 5.Syncopal episodes
2.Complex partial seizure The patient displays loss of consciousness/awareness with motor automatisms like lip-smacking and hand-clasping characteristic of a complex partial seizure. The patient has had multiple episodes of staring, lip-smacking, and hand-clasping characteristic of a complex partial seizure, most likely originating in the temporal lobe. Simple partial seizures do not result in a loss of consciousness while complex partial seizures do (though the patient may appear awake). Patients with a complex partial seizure also have a postictal state during which they are confused and may be lethargic or tired for many hours after the seizure. Incorrect Answers: Answer 1: The patient has a loss of consciousness and awareness characterized by non-responsiveness to his parents' voice making this a complex seizure. Answer 3: A generalized tonic-clonic seizure is characterized by global myotonic/myoclonic jerks across multiple extremities. This patient's staring and automatisms are more characteristic of a complex partial seizure. Answer 4: Absence seizures involve staring in space as well, but are usually a few seconds long, can happen multiple times a day, and don't have a post-ictal state. Answer 5: While jerking can happen during a syncopal episode, this patient's automatisms and postictal confusion are more characteristic of a seizure. Summary: Partial seizures are focal in nature and can be either simple (no loss of consciousness) or complex (with loss of consciousness).
A 62-year-old female is being evaluated in the emergency department for worsening agitation as well as frequent mood swings. According to her daughter, she has been developing progressive forgetfulness and has frequently been getting lost on her way back home while driving on familiar routes. She also had brief, but frequent periods of confusion characterized by nonsensical speech. Other reported symptoms include excessive daytime sleepiness as well as difficulty sleeping during the night. Over the past twelve months, the patient has had increasing visual hallucinations and paranoid behavior accompanied by agitation and restlessness. Her medical history is significant for mild depression, and her only medication is sertraline. A depression screen was negative for a depressed mood. On physical examination, the temperature is 37°C (98.6°F), blood pressure is 125/80 mm Hg, pulse rate is 78/min, respiratory rate is 18/min, and oxygen saturation is 96% on room air. General physical examination is unremarkable. Neurologic examination shows an agitated and restless female with masked facies, a soft voice, postural instability, and a slow gait. She scores 24/30 on the mini-mental state examination (MMSE), where she lost points on orientation to time and place, delayed recall, and figure drawing. The patient receives haloperidol in the emergency department to treat the agitation, which results in worsening of her symptoms with associated limb and neck stiffness. Which of the following is the most likely diagnosis? 1.Alzheimer dementia 2.Dementia with Lewy bodies 3.Bipolar disorder with psychosis Major depression with psychosis
2.Dementia with Lewy bodies This patient most likely has dementia with Lewy bodies, which is the second most common cause of dementia. The classic findings of dementia with Lewy bodies include fluctuating attention and alertness; recurrent, well-formed visual hallucinations; and spontaneous parkinsonism. Sensitivity to neuroleptic medications such as haloperidol is commonly seen. The patient's presenting symptoms, as well as associated sensitivity to haloperidol, all point towards the possibility of dementia with Lewy bodies. Sensitivity to neuroleptic medications is less common in Alzheimer disease compared with Lewy body dementia. Delirium typically has a more rapid onset, unlike this patient where symptoms have been present for a year. Bipolar disorder with major depression is less likely in this patient with the above constellation of findings.
A 62-year-old male is brought to his PCP by his wife because she is concerned that he has become more confused over the past month. Specifically, he has been having difficulty finding words and recently started forgetting the names of their friends. She became particularly worried when he got lost in their neighborhood during a morning walk. Finally, he has had several episodes of incontinence and has tripped over objects because he "does not lift his feet off the ground" while walking. He has a history of hypertension and diabetes but has otherwise been healthy. His family history is significant for many family members with early onset dementia. Which of the following treatments would most likely be effective for this patient? 1.Better control of diabetes and hypertension 2.Galantamine 3.Placement of shunt 4.Selegiline Tetrabenazine
3.Placement of shunt This patient with dementia, incontinence, and hypokinetic or "magnetic" gait most likely has normal pressure hydrocephalus, which can be treated by placing a shunt in the cerebral ventricles. Normal pressure hydrocephalus presents with a classic triad of dementia, gait disturbances, and urinary incontinence. The gait disturbance is classically described as a "magnetic" gait where the feet remain "stuck to the ground". Importantly, normal pressure hydrocephalus is a potentially reversible cause of dementia and can be diagnosed through observation of pathologically enlarged ventricles in the setting of a normal opening pressure noted on lumbar puncture. Placement of a shunt to drain extra fluid can often reverse the condition. Incorrect Answers: Answer 1: Better control of diabetes and hypertension is used in order to ameliorate the effects of vascular dementia. It would not be used in normal pressure hydrocephalus. Answer 2: Galantamine is an acetylcholinesterase inhibitor that is used in the treatment of Alzheimer disease. It would not be used in normal pressure hydrocephalus. Answer 4: Selegiline is a monoamine-oxidase type B inhibitor that is used in the treatment of Parkinson disease. It would not be used in normal pressure hydrocephalus. Answer 5: Tetrabenazine increases the depletion of amines such as dopamine and can be used in the treatment of Huntington disease. It would not be used in normal pressure hydrocephalus. Summary: Normal pressure hydrocephalus presents with dementia, magnetic gait, and urinary incontinence.
A 24-year-old college student consumed a container of canned vegetables for dinner. Fourteen hours later, he presents to the E.R. complaining of difficulty swallowing and double-vision. The bacterium leading to these symptoms is: 1.An obligate aerobe 2.Gram-negative 3.Rod-shaped 4.Non-spore forming 5.Cocci-shaped
3.Rod-shaped The patient described above is suffering from a Clostridium botulinum infection due to the consumption of the uncooked canned food. The consumption of uncooked canned food, smoked fish, and reheated rice are common routes of botulism toxin (produced by C. botulinum) into the body. C. botulinum produces a neurotoxin (botulism toxin) that can bind to motor neurons. This binding and entry into motor neurons leads to the inhibition of vesicle (packaged with neurotransmitters) fusion; therefore, release of ACh is impaired and there is subsequent paralysis. This leads to symptoms such as paralysis, dysphagia, and diplopia.
A 66-year-old well-respected male cardiologist is referred by the hospital review board for psychiatric evaluation in a neighboring county after he was accused of improper sexual advances by several female patients. During the evaluation, the cardiologist has good cognitive function, but he is less verbal and when he speaks, he is reckless. He denies all the accusations. He acknowledges several episodes of his recent uncharacteristic, public verbal attacks on junior colleagues. He has continued to work, maintain family life, and pursue his hobbies. A neurologist who previously examined the cardiologist has already reported that his MMSE score was 26/30. Physical examination is unremarkable. MRI of the brain and an EEG show no abnormalities either. Which of the following is the most likely diagnosis? 1.Mild cognitive impairment 2.Alzheimer disease 3.Lewy bodies dementia 4. Frontotemporal dementia
4. Frontotemporal dementia The patient has shown uninhibited behavior and impaired judgment, inappropriate sexual advances, and recklessness. Some of his behavior might have been aggressive. He does not fulfill criteria for dementia because he can still function at work and home, and his MMSE score, while lower than expected for his education, is within normal limits. Although the cardiologist does not have dementia, he is not normal. He has most likely developed frontotemporal dementia due to above-described findings.
A 78-year-old female presents to the emergency department with sudden onset neurologic symptoms. She states that she suddenly felt "strange", thus prompting her presentation. She endorses an ongoing headache. The patient has a past medical history of hypertension, diabetes, and atherosclerosis. Her temperature is 97.6°F (36.4°C), blood pressure is 177/91 mmHg, pulse is 90/min, respirations are 15/min, and oxygen saturation is 98% on room air. Physical exam is notable for numbness on the patient's left lower face. The patient's headache improves with the administration of acetaminophen. She has decreased pain and temperature sensation on the right side of her body. She demonstrates an ataxic gait when walking to the bathroom. Her gag reflex is not present as well. Which of the following is the most likely diagnosis? 1.Anterior cerebral artery territory infarct 2.Hemiplegic migraine 3.Middle cerebral artery territory infarct 4.Posterior inferior cerebellar artery territory infarct 5.Vertebrobasilar insufficiency
4. Posterior inferior cerebellar artery territory infarct This patient is presenting with sudden onset numbness on her left lower face, decreased pain/temperature sensation on the right side of her body, ataxia, and an absent gag reflex, which are concerning for lateral medullary syndrome (Wallenberg syndrome). This caused by a left posterior inferior cerebellar artery territory infarct in this patient. Lateral medullary syndrome occurs when there is an infarct of the territory supplied by the posterior inferior cerebellar artery (PICA). Presenting symptoms include a contralateral loss of pain/temperature sensation (from the involvement of the spinothalamic tract), a central Horner syndrome that is ipsilateral (from descending hypothalamic tract involvement), and an absent gag reflex with trouble swallowing caused by dysfunction of cranial nerves IX and X. Lateral medullary syndrome can cause vertigo. The vertigo of a cerebellar infarct is constant, results in ataxia, does not change with changes in head position as compared to peripheral causes of vertigo, and is generally more severe when compared to vestibular neuritis/labyrinthitis. Incorrect Answers: Answer 1: Anterior cerebral artery territory infarct will present with notable lower extremity weakness (contralateral to the lesion), mild upper extremity weakness (contralateral), personality changes, and urinary incontinence. Answer 2: Hemiplegic migraine presents with a headache and stroke-like symptoms which may include slurring of a patient's speech, focal weakness, or visual changes. It is paramount to consider stroke in these patients; however, a notable feature of a hemiplegic migraine is neurological symptoms that improve as the headache improves. Answer 3: Middle cerebral artery territory infarct presents with profound upper extremity weakness (contralateral), speech deficits (aphasia or dysarthria, common in a left middle cerebral artery infarct), and hemineglect (in particular when a right middle cerebral artery infarct). Answer 5: Vertebrobasilar insufficiency presents with vertigo, nausea/vomiting, syncope, nystagmus, dysarthria, dystonia, and an ataxic gait. Findings are often bilateral in vertebrobasilar artery insufficiency.
A 73-year-old male presents with sudden onset hemiplegia. He has no known significant medical history. He becomes progressively lethargic. Emergency head CT results are shown. Neurosurgery is consulted, but the patient expires. An autopsy is performed; a section from that autopsy is shown. Which of the following was the predisposing cause? 1.Vascular malformation 2.Vasculitis 3.Hypertension 4.Amyloid angiopathy 5.Cardiac arrhythmia
4.Amyloid angiopathy This patient had a spontaneous lobar hemorrhage as seen on CT scan. The most common cause of spontaneous lobar hemorrhage in the elderly is amyloid angiopathy. Intracerebral hemorrhage (ICH) is the second leading type of stroke after ischemic stroke. The most common overall cause of ICH is hypertension. However, most hypertensive hemorrhages occur in the deep areas of the brain, including the putamen, thalamus, and pons. The most common cause of lobar hemorrhage (hemorrhage in the cortex and subcortical white matter) is amyloid angiopathy. The presentation is highly variable, depending on the size of the hemorrhage and vascular territory involved. Larger hemorrhages frequently present with hemiplegia and altered consciousness, as in this question stem. Incorrect Answers: Answers 1-2: Vascular malformation and vasculitis are a less common cause of ICH and are made even less likely here because of the histological finding of amyloid angiopathy. Answer 3: Hypertension is the most common cause of ICH but tends to occur in deep regions of the brain. Additionally, the histologic findings of amyloid angiopathy in make hypertension less likely. Answer 5: Atrial fibrillation can predispose to embolic ischemic stroke.
A 14-year-old female notes that while her fever, malaise, and a sore throat have dissipated a week ago, her hacking cough persists. A chest radiograph demonstrates a streaky infiltrate in her right lung that appears much worse than the symptoms she is experiencing. A cold agglutination test conducted on her blood shows clumping together when placed at 4 degrees Celsius. Which of the following medications would best treat the agent causing her cough? 1.Vancomycin 2.Isoniazid 3.Ampicillin 4.Azithromycin 5.Pyrazinamide
4.Azithromycin The clinical presentation is indicative of Mycoplasma pneumoniae. The first-line antibiotic to treat M. pneumoniae infection is azithromycin. Mycoplasma pneumoniae, which lack a cell wall, are the smallest bacteria capable of self-replication. Beta lactam antibiotics, such as penicillin, ampicillin, and ceftriaxone, that target the peptidoglycan cell wall of gram-positive bacteria, are ineffective against M. pneumoniae. Antibiotics that target the various protein machinery within M. pneumoniae are effective at targeting this bacterium - doxycycline (binds to 30S ribosomal subunit), azithromycin (binds the 50S ribosomal subunit), and ciprofloxacin (inhibits the bacterial topoisomerases). The chest radiograph of an infection with M. pneumoniae will tend to appear worse than the symptoms the patient is experiencing. Answer 1: Vancomycin is a glycopeptide that prevents the synthesis of the peptidoglycan wall by binding to the D-alanyl-D-alanine end of the cell wall.Answer 2: Isoniazid is used to treat M. tuberculosis by inhibiting mycolic acid synthesis.Answer 3: Ampicillin inhibits transpeptidases which prevent the synthesis of the cell wall.Answer 5: Pyrazinamide is also used to treat M. tuberculosis by inhibiting mycobacterial fatty-acid synthetase I.
A 32-year-old farmer is brought to the emergency department by his wife. The patient was reportedly anxious, sweaty, and complaining of a headache and chest tightness before losing consciousness in route to the hospital. Which of the following is mechanistically responsible for this patient's symptoms? 1.Competitive inhibition of acetylcholine at post-junctional effector sites 2.Binding of acetylcholine agonists to post-junctional receptors 3.Inhibition of presynaptic exocytosis of acetylcholine vesicles 4.Irreversible inhibition of acetylcholinesterase 5.Reversible inhibition of acetylcholinesterase
4.Irreversible inhibition of acetylcholinesterase Normally, AChE rapidly converts acetylcholine (ACh) into inactive metabolites (choline and acetate) in the synaptic cleft. In organophosphate poisoning, over-stimulation of the post-junctional acetylcholine receptor occurs due to irreversible inhibition of AChE by organophosphate compounds. The cholinergic toxicity associated with this poisoning is managed by administering atropine and pralidoxime. Atropine prevents cholinergic activation by competing with acetylcholine at muscarinic receptors, and not nicotinic receptors. Pralidoxime reactivates cholinesterase, thus having a favorable effect at the level of nicotinic and muscarinic receptors. These medications are given concurrently. Incorrect Answers: Answer 1: Various medications, such as atropine and scopolamine, antagonize ACh at nicotinic, muscarinic, and neuromuscular receptors to selectively block ACh activity. Answer 2: ACh agonists such as pilocarpine and cevimeline are used for the treatment of open angle glaucoma and Sjogren's syndrome, respectively. Answer 3: Inhibition of presynaptic exocytosis of ACh vesicles is the mechanism of botulinum neurotoxin. It results in decreased activity of ACh. Answer 5: Reversible inhibition of AChE is the mechanism employed by many medications including those used to treat Alzheimer's disease (donepezil) and myasthenia gravis (pyridostigmine). The danger of organophosphates is their IRREVERSIBLE inhibition of AChE.
A 65-year-old male presents to the ER after an accident with his bicycle 3 hours ago. He explains that he was riding in the woods and fell on his left side. After getting back up, the patient noticed he had difficulty hearing on his left side and decided to visit the hospital. Vitals are unremarkable. The patient is alert and oriented and demonstrates understanding of his situation. On his visual exam, the patient's visual fields are intact. When the patient asked to smile, it is asymmetric, and his left forehead is not wrinkled. On otoscope, there is a dark fluid pushing on his left tympanic membrane. The patient is also unable to walk in a straight line without feeling unsteady. The patient is admitted for observation. A day later, bruising over his left mastoid process is noted. Which of the following is the most likely cause of the patient's symptoms? 1.Anterior cranial fossa fracture 2.Occipital skull fracture 3.Orbital roof fracture 4.Temporal bone fracture 5.Zygomaticofrontal fracture
4.Temporal bone fracture This patient is presenting for a trauma secondary to falling off his bicycle, leading to hearing loss, periauricular ecchymosis (bruising over mastoid process), facial paralysis (asymmetric smile and unwrinkled forehead), hemotympanum (blood in the tympanic cavity), and dizziness. This constellation of symptoms is caused by a temporal bone fracture. Temporal bone fractures often result from blunt force trauma, such as motor vehicle accidents or falls. Located on the lateral sides of the skull, the temporal bones house critical structures such as the cochlea, vestibulocochlear nerve (CN8), and facial nerve (CN7). Hearing loss, the most common symptom, as well as dizziness occur due to damage to the cochlea and CN8. Facial paralysis is secondary to CN7 damage. Periauricular ecchymosis, also known as Battle sign, is due to extravasation of blood following the course of the posterior auricular artery and indicates damage to the middle cranial fossa. Diagnosis is confirmed by CT scan, where the fracture is directly visualized. Surgical intervention is often necessary for neurological symptoms, but otherwise expectant management occurs. Incorrect Answers: Answer 1: Anterior cranial fossa fracture could damage the orbit and cribriform plate, possibly leading to periorbital ecchymosis, CSF otorrhea, and/or anosmia. Answer 2: Occipital skull fracture could damage the occipital lobe, leading to loss of vision in the side contralateral to the site of injury (e.g., left occipital lobe damage causing loss of right visual field of both eyes). Answer 3: Orbital roof fracture can lead to subconjunctival hemorrhage, periorbital hematoma as well as visual problems. With severe injury leading to brain herniation, pulsating proptosis may be observed. Answer 5: Zygomaticofrontal fracture is caused by a direct insult to the malar eminence of the cheek. It can present with periorbital ecchymosis, loss of cheek projection, and cheek and upper lip sensation loss due to infraorbital nerve damage.
A 72-year-old male with longstanding history of diabetes mellitus and hypertension presents to the emergency department with sudden-onset numbness. On your neurological exam, you note that he has loss of sensation on the left side of his face, arm, and leg. His motor strength exam is normal, as are his cranial nerves. Which of the following is the most likely explanation for his presentation? 1.Anterior cerebral artery stroke 2.Middle cerebral artery stroke 3.Conversion disorder 4.Thalamic stroke Basilar artery stroke
4.Thalamic stroke This patient had a stroke with total sensory loss, consistent with thalamic stroke. Lacunar strokes occur in areas of the brain that are supplied by small perforating vessels. Risk factors for lacunar stroke include atherosclerosis, hypertension, and diabetes. These strokes arise due to hyaline arteriolosclerosis of perforating vessels, and typically affect the deep nuclei of the brain, including the caudate, thalamus, and putamen. Incorrect Answers: Answer 1: Anterior cerebral artery stroke usually presents with contralateral loss of sensation and motor function worse in the lower limb. Answer 2: Middle cerebral artery stroke may present with contralateral loss of sensation and motor function worse in the upper limb, and/or may present with aphasia if the dominant hemisphere is involved. Answer 3: Conversion disorder is a rare condition in which neurological symptoms rapidly appear without apparent organic cause. Answer 5: Basilar artery stroke can present with motor symptoms, altered consciousness, headache, and visual disturbances.
A 49-year-old female is brought to the hospital for evaluation after being arrested for shoplifting several items of trivial value from a convenience store. She has no prior medical or psychiatric history and reports feeling "fine." She is dressed provocatively and makes sexual remarks to the physician examining her. She discloses that she was fired from her job 1 month ago after missing several weeks of work but states "None of this matters. What do I care?" The patient's husband arrives at the hospital and expresses concerns about her saying she has suddenly become hypersexual and impulsive, spending money excessively and binge eating over the last 2 months. While in the examination room, the patient grabs several bandages and cotton balls, puts them in her mouth, and urinates on the examination table. Her affect is blunted. The remainder of the psychiatric mental status exam, neurological exam, and physical exam are normal. Urine toxicology is negative. What is the most likely diagnosis in this patient? 1.Anti-NMDA receptor encephalitis 2.Bipolar I disorder 3.Borderline personality disorder 4.Dementia with Lewy bodies 5. Pick disease (frontotemporal dementia)
5. Pick disease (frontotemporal dementia) This patient with 2 months of behavioral disinhibition, apathy, and hyperorality, with poor insight, has frontotemporal dementia. Frontotemporal dementia (FTD or Pick disease) is a group of neurodegenerative disorders that primarily affects the frontal and temporal lobes. It is one of the most common causes of early-onset dementia (average age of onset is in the 50s) and presents with early personality and behavioral changes (behavioral variant FTD, the most common form) or early aphasia (primary progressive aphasia). Memory and visuospatial function are preserved (unlike most other types of dementia, especially Alzheimer dementia), and there are no motor abnormalities early in the disease; thus, behavioral variant FTD is often confused for a psychiatric disorder. The most common early signs of behavioral variant FTD are behavioral disinhibition (e.g., socially inappropriate sexual behaviors and public urination), with impulsive or compulsive behaviors, apathy (mimicking depression), and hyperorality (increased eating and placement/consumption of inedible objects in mouth). Incorrect Answers: Answer 1: Anti-NMDA receptor encephalitis is a subacute neurological disorder that presents with psychiatric manifestations, especially impulsive and bizarre behaviors (like those of this patient). However, dyskinesias are also typically seen, as well as autonomic instability, decreased consciousness, and/or seizures. Answer 2: In bipolar I disorder, a manic episode can cause excessive engagement in pleasurable activities (e.g., hypersexuality and excessive spending). However, decreased need for sleep, grandiosity, decreased appetite/weight loss (rather than binge eating), pressured speech, flight of ideas, and/or psychomotor agitation are also seen, and this patient's normal mood and blunted affect make mania implausible (mean onset age = 21). Answer 3: Borderline personality disorder could also explain some of this patient's provocative and impulsive behavior, but, like all personality disorders, is lifelong and would not begin at age 49. Answer 4: In dementia with Lewy bodies, as in frontotemporal dementia, frank memory impairment occurs later in the disease. Earlier features are psychiatric manifestations (hallucinations, delusions, apathy, and/or depression) and parkinsonism (e.g., bradykinesia, tremor, cogwheeling rigidity, and shuffling gait).
A 78-year-old male has been hospitalized for the past 3 days after undergoing a revision left total hip replacement. Over the past several hours, the nursing staff reports that the patient has exhibited fluctuating periods of intermittent drowsiness and confusion where he has been speaking to nonexistent visitors in his hospital room. The patient's daughter is present at bedside and reports that the patient lives alone and successfully manages his own affairs without assistance. Which of the following is most likely true of this patient's current condition? 1.Anticholinergic medications may alleviate his symptoms 2.Beta-amyloid plaques and neurofibrillary tangles are pathologic findings associated with this condition 3.Short-term memory is often impaired, with sparing of remote memory 4.The condition is typically irreversible, representing a common complication of aging 5.Possible etiologies include infection, trauma, or polypharmacy
5.Possible etiologies include infection, trauma, or polypharmacy This patient's presentation is most consistent with delirium, which has numerous potential etiologies including infection, surgery or other trauma, or pharmacologic side effects.Acute onset of symptoms, fluctuating level of consciousness, and presence of visual hallucinations favor a diagnosis of delirium over dementia. Additionally, sleep-wake cycle disturbance is a prominent feature of delirium. Delirium is reversible, while dementia, by contrast, is often irreversible and is typically a diagnosis of exclusion confirmed by histology on autopsy. Clinically, patients with dementia show gradual decline in cognition with preservation of level of consciousness. Additional findings in dementia include aphasia, apraxia, agnosia, behavioral or personality changes, and short-term memory impairment with sparing of remote memory. Incorrect Answers: Answer 1: Anticholinergic medications may exacerbate delirium. Answer 2: Beta-amyloid plaques and neurofibrillary tangles are common findings in Alzheimer's dementia, not delirium. Answer 3: This is a clinical manifestation of dementia; delirium, in contrast, is more commonly associated with global memory impairment. Answer 4: Delirium is generally a reversible process once an underlying etiology is identified and treated; dementia, on the other hand, is typically irreversible, however, it is important to note that it is NOT a component of the normal process of aging.
A 15-year-old male is brought to the ER after he passed out in the hallway. On presentation, he is alert but confused about why he is in the hospital. He says that he remembers seeing flashes of light to his right while walking out of class but cannot recall what happened next. His next memory is being woken up by emergency responders who wheeled him into an ambulance. A friend who was with him at the time says that he seemed to be swallowing repeatedly and staring out into space. He has never had an episode like this before, and his past medical history is unremarkable. Which of the following characteristics is most likely true of the cause of this patient's symptoms? 1.Begins with 10-15 seconds of muscle contraction 2.Demonstrates quick and repetitive jerks of extremities 3.Episodes with 3-4 hertz spike and wave discharges 4.Isolated to the left occipital lobe 5.Starts in the left occipital lobe and then generalizes
5.Starts in the left occipital lobe and then generalizes This patient who saw visual flashes and then lost consciousness while demonstrating automatisms most likely had a partial non-motor seizure with impaired awareness that started in the left occipital lobe and secondarily generalized to the temporal lobe to produce oral automatisms. Focal seizures are episodes of abnormal neuronal activity in a localized part of the brain. The initial symptoms experienced by patients with focal seizures will therefore depend on the anatomical location in which the seizure started. For example, patients with a seizure that starts in the primary visual cortex can see abnormal shapes or flashes. If the patient were to develop oral automatisms, then it suggests the seizure is involving the mesial (medial) temporal lobe. Patients who develop focal seizures with preservation of consciousness are said to have focal seizures with awareness. If patients have a loss of consciousness during the focal seizure, then they are said to have focal seizures with impaired awareness. Incorrect Answers: Answer 1: Begins with 10-15 seconds of muscle contraction is characteristic of tonic-clonic seizure; however, this patient's initial visual symptoms and subsequent automatisms are more characteristic of complex partial seizures. Answer 2: Demonstrates quick and repetitive jerks of extremities is characteristic of myoclonic seizures; however, this patient's initial visual symptoms and subsequent automatisms are more characteristic of complex partial seizures. Answer 3: Episodes with 3-4 hertz spike and wave discharges is characteristic of absence seizures, which can be sometimes associated with automatisms but not with visual symptoms. This patient's initial visual symptoms are more characteristic of complex partial seizures. Answer 4: Isolated to the left occipital lobe is characteristic of simple partial seizures; however, this patient's loss of consciousness and subsequent automatisms are more characteristic of complex partial seizures. Summary: Complex partial seizures start in a focal location but also lead to impaired consciousness and automatisms.
A 44-year-old female is admitted to the neurological service. You examine her chart and note that after admission she was started on nimodipine. Which of the following pathologies would benefit from this pharmacologic therapy? 1.Pseudotumor cerebri 2.Thromboembolic stroke 3.Epidural hematoma 4.Subdural hematoma 5.Subarachnoid hemorrhage
5.Subarachnoid hemorrhage Nimodipine, which is a calcium-channel blocker (CCB), is used in the treatment of subarachnoid hemorrhage to reduce post-bleed vasospasm and further ischemia to the brain. The most common cause of subarachnoid hemorrhages is an aneurysmal rupture (commonly berry aneurysms). Patient populations at risk include patients with Marfan syndrome, Ehlers-Danlos syndrome, adult-polycystic kidney disease (ADPKD) or an arteriovenous malformation (AVM). The classical presentation is the "worst headache of my life." Patients should be evaluated with a non-contrast head CT as well as perhaps an LP (looking for a bloody/xanthochromic tap). Treatment involves evacuation of blood/lower intracranial pressure and administration of calcium channel blockers. CCBs reduce the risk of vasospasm following breakdown of blood 2-3 days post rupture. Incorrect Answers: Answer 1: Pseudotumor cerebri may be improved with blood pressure control but there is no specific role for nimodipine. Answer 2: Thromboembolic stroke outcomes may be improved with "clot busters" but there is no specific role for nimodipine. Answer 3: Epidural hematomas may be improved with surgical evacuation but there is no specific role for nimodipine. Answer 4: Subdural hematomas may be improved with surgical evacuation but there is no specific role for nimodipine.
A 45-year-old male arrives by ambulance to the emergency room after being involved in a very severe construction accident. Upon arrival to the ER, the patient was unconscious and unresponsive. His vital signs are BP: 80/40, HR: 120 bpm, RR: 25 bpm, Temperature: 97.1 degrees, and SPO2: 99%. During surgery, the patient had an acute drop in his blood pressure. ST elevations were noted in multiple leads. This resolved with adequate fluid resuscitation and numerous blood transfusions. On his third day in the ICU, he experienced an oxygen desaturation of 85% despite being on a respirator with 100% oxygen at 15 breaths/minute. On auscultation, air entry is present bilaterally with the presence of crackles. A 2/6 systolic murmur is heard. Readings from a Swan-Ganz catheter display the following: central venous pressure (CVP): 4 mmHg, right ventricular pressure (RVP) 20/5 mmHg, pulmonary artery pressure (PAP): 20/5 mmHg. Pulmonary capillary wedge pressure (PCWP): 5 mm Hg. A chest x-ray is shown. The patient dies soon after this episode. What is the most likely direct cause of his death? A.Diffuse alveolar damage B.Ventricular septal defect C.Myocardial free wall rupture D.Papillary muscle rupture E.Myocardial reinfarction
A.Diffuse alveolar damage This individual is likely suffering from Acute Respiratory Distress Syndrome (ARDS) which is classified histologically as diffuse alveolar damage (DAD). Acute respiratory distress syndrome is an acute onset bilateral patchy airspace disease which results in noncardiogenic pulmonary edema and severe hypoxemia. The criteria for diagnosis require an acute onset (generally less than 24 hours) of hypoxemia with bilateral infiltrates on chest radiographs. Additionally, there must be no evidence of increased left atrial pressure (PCWP <18), and severe hypoxemia (PaO2/FiO2 less than or equal to 200). Answers 2-5: The rest of the answer choices could result in similar signs and symptoms (pulmonary edema, hypoxemia) but would result in an increased PCWP (>18 mmHg) because they are the result of acute heart failure.
A 38-year-old African American female presents to her primary care physician with uveitis, cough, and arthralgias in her ankles and legs. Bloodwork reveals elevated angiotensin converting enzyme levels, and skin PPD testing shows no observable induration after 48 hours. The patient demonstrates reduced FEV1 and FVC upon spirometry. FEV1/FVC is 85%. Which of the following would you expect to see upon chest X-ray? A.Enlarged hilar lymph nodes B.Kerley B Lines C.Bilateral diaphragmatic elevation. D.Pleural effusion Fluid in alveolar walls
A.Enlarged hilar lymph nodes The clinical presentation is consistent with sarcoidosis. Enlarged hilar lymph nodes are commonly seen on chest radiographs of patients with sarcoidosis.Sarcoidosis is a syndrome involving granuloma formation in multiple organs, namely the lungs, liver, spleen, and bones. It is most found in African American females and results in symptoms such as uveitis, cough, and arthralgias. Spirometry findings in these patients are consistent with restriction, though total lung capacity would need to be reduced to confirm restriction. Spirometry in sarcoidosis can also show obstruction or a mixed pattern. Incorrect Answers:Answers 2: Kerley B lines are a sign of interstitial pulmonary edema.Answer 3: Bilateral diaphragmatic elevation can be seen in obesity (low lung volumes from increased intra-abdominal girth and workload) or neuromuscular disease as well as other conditions would be likely show restriction of spirometry, however it would not be expected in sarcoidosis.Answer 4: Although pleural effusion resulting in fluid in the alveolar walls is a rare complication of sarcoidosis, it is much less common than enlarged hilar lymph nodes.Answer 5: Fluid in the alveolar walls typical of pulmonary edema is not commonly found in sarcoidosis. Rather, sarcoidosis can cause pulmonary arterial hypertension, which can result in right heart failure and peripheral edema.
A 54-year-old female presents to the ED complaining of shortness of breath. Her temperature is 100.1°F (37.8°C), blood pressure is 130/85 mmHg, pulse is 105/min, and respirations are 24/min. Physical examination reveals bilateral rales and dullness to percussion at the lung bases that is worse on the left. Hepatosplenomegaly is noted. A chest radiograph is shown. A thoracentesis and hematologic analysis are performed, with the following results: Pleural lactate dehydrogenase: 54 U/L Serum lactate dehydrogenase: 82 U/L Proteins, pleural fluid: 3.8 g/dL Proteins, serum (total): 7.0 g/dL Which of the following conditions is most strongly associated with these findings? A.Pulmonary embolism B.Cirrhosis C.Nephrotic syndrome D.Protein losing enteropathy E.Congestive heart failure
A.Pulmonary embolism The patient in this vignette presents with dyspnea, tachypnea, dullness to percussion, radiographic blunting of the costophrenic angle, and lab values suggestive of an exudative effusion, which could be caused by a pulmonary embolism. Pleural effusions represent accumulations of fluid in the pleural space. Effusions are classified depending on their underlying etiology as either transudative (due to increased pulmonary capillary wedge pressure or decreased oncotic pressure) or exudative (due to increased vascular permeability). Light's criteria can be used to differentiate transudative and exudative effusions. Per Light's criteria, a effusion is exudative if it satisfies at least one of the following: 1.pleural/serum protein ratio > 0.5 ● 2.pleural/serum LDH > 0.6 ● 3.pleural LDH greater than 2/3 the normal upper limit for serum LDH Picture demonstrates the characteristic appearance of a unilateral pleural effusion. Note the blunted costophrenic angle on the left.
A 73-year-old female is hospitalized following a pelvic fracture. She undergoes surgical repair without complication. Four days into her hospital stay, she develops acute dyspnea and chest pain accompanied by oxyhemoglobin desaturation. Which of the following arterial blood gas values is the patient most likely to have? (normal values: pH 7.35 - 7.45, PaO2 80 - 100 mm Hg, PaCO2 35-45 mm Hg, HCO3 22-26) A.pH 7.5, PaO2 60, PaCO2 30, HCO3 22 B.pH 7.3, PaO2 60, PaCO2 30, HCO3 20 C.pH 7.5, PaO2 60, PaCO2 50, HCO3 28 D.pH 7.3, PaO2 60, PaCO2 50, HCO3 24 E.pH 7.4, PaO2 60, PaCO2 40, HCO3 24
A.pH 7.5, PaO2 60, PaCO2 30, HCO3 22 The patient in this vignette most likely has a pulmonary embolism (PE) secondary to a deep vein thrombosis (DVT). Classically, a PE results in a hypoxic, respiratory alkalosis. Advanced age, immobilization, and a pelvic fracture are all risk factors for development of a deep vein thrombosis in this patient. Symptoms of PE include sudden-onset dyspnea, pleuritic chest pain, low-grade fever, cough, anxiety, and (rarely) hemoptysis. Physical exam can show tachypnea, tachycardia and hypoxia. Signs of DVT include an erythematous, swollen, and warm lower extremity. Characteristically, a PE will increase dead space resulting in a ventilation-perfusion mismatch (V/Q = infinity). Incorrect Answers:Answer 2: Low pH with a low PaCO2 is more consistent with metabolic acidosis with respiratory compensation.Answer 3: High pH with a high PaCO2 is more consistent with a metabolic alkalosis with respiratory compensation.Answer 4: Low pH with a high PaCO2 is more consistent with a respiratory acidosis.Answer 5: Normal pH with normal CO2 is not physiologically possible with low PaO2.
A 56-year-old female presents to the emergency department complaining of abdominal pain. She reports a three-hour history of severe epigastric pain that radiates to her back. Her past medical history is notable for hypertension, diabetes, and alcoholism. She takes enalapril and metformin. Initial laboratory evaluation is notable for leukocytosis, elevated lipase, and elevated serum LDH. She is admitted to the hospital and given fluids and pain control. However, she develops shortness of breath on the second day of her admission. Her temperature is 99°F (37.2°C), blood pressure is 140/85 mmHg, pulse is 110/min, respirations are 24/min, and SpO2 is 90% on room air. Physical exam is notable for increased work of breathing with rales at the bilateral lung bases. A chest radiograph is shown in Figure A. Electrocardiogram reveals normal sinus rhythm. Brain natriuretic peptide is normal. An echocardiogram reveals no cardiomegaly. Which is the following findings is most likely to be seen in this patient? A.Fat globules in the pulmonary capillaries B.Intra-alveolar hyaline membranes C.Acute inflammatory infiltrate with intra-bronchial neutrophils D.Hemosiderin-laden macrophages Wedge-shaped fibrinous mass consisting of interdigitating areas of pink and red
B.Intra-alveolar hyaline membranes The patient in this vignette presents with dyspnea and hypoxia while being treated for acute pancreatitis, which is suggestive of acute respiratory distress syndrome (ARDS). Intra-alveolar hyaline membranes can be seen in patients with ARDS. ARDS causes acute onset of respiratory failure, cyanosis, and hypoxia. Various conditions like sepsis, pancreatitis, pulmonary infections, aspiration, uremia, and trauma all lead to this common pathway of damage to the respiratory endothelial lining. The damaged lining then increases alveolar permeability and leakage of protein-rich fluid into the alveoli forming intra-alveolar hyaline membranes. Incorrect Answers: Answer 1: Fat embolism syndrome occurs when fat emboli from the exposed bone marrow following a long bone fracture lodge in the pulmonary arterial vasculature. Clinically, it presents with the triad of neurologic impairment, respiratory distress, and a petechial rash following a long bone fracture. Answer 3: Pneumonia is defined as an infection of the lung parenchyma. It is often divided into bronchopneumonia and lobar pneumonia. Bronchopneumonia is characterized by an acute inflammatory infiltrate in the bronchi or bronchioles. Lobar pneumonia is characterized by congestion, neutrophils, and fibrinopurulent exudate within some of the lung lobes. Answer 4: Hemosiderin-laden macrophages (also known as heart failure cells) form in the alveoli of patients with heart failure or chronic pulmonary edema. They arise because the backup of blood from the heart into the alveolar capillaries forces red blood cells into the lung parenchyma where they are then engulfed by alveolar macrophages. Over time, these macrophages become filled with hemosiderin. A normal brain natriuretic peptide and absence of cardiomegaly in this patient suggest that heart failure is less likely than ARDS. Answer 5: Pulmonary emboli most often occur because clots from the deep leg veins embolize and traverse the venous tracts into the pulmonary vasculature. Thrombi are characterized by interdigitating areas of pale Picture demonstrates the characteristic appearance of ARDS. Note the patchy bilateral alveolar infiltrates stemming from increased alveolar permeability.
A 34-year-old male is brought to the emergency department with a 1-hour history of increasing shortness of breath. He says that he was smoking outside during a break at work when he started having difficulty breathing. He has smoked 2 packs of cigarettes per day since he was 14 years old and has a childhood history of asthma. He works as a contractor and primarily replaces walls in old houses. His family history is significant for autoimmune diseases as well as lung cancer. Histology reveals the findings shown. Which of the following is the most likely cause of this patient's symptoms? A.Asbestosis B.Asthma C.Chronic obstructive pulmonary disease D.Lung cancer Sarcoidosis
B.Asthma This patient who presents with acute onset shortness of breath and is found to have Charcot-Leyden crystals on histology is most likely suffering from an asthma exacerbation. Asthma is a disease characterized by episodic and reversible bronchoconstriction that is caused by a type 1 hypersensitivity reaction to an inhaled antigen. Patients will present with shortness of breath, tachycardia, and bilateral expiratory wheezing. Pulmonary function tests will reveal an obstructive pattern with a decreased forced expiratory volume in the first second (FEV1) to forced vital capacity (FVC) ratio. Unlike in chronic obstructive pulmonary disease, this decrease in peak flow should be reversible with administration of albuterol (a beta-agonist that leads to bronchodilation) and exacerbated with administration of methacholine (a cholinergic agonist that leads to bronchoconstriction). Histology will reveal Charcot-Leyden crystals that are composed of aggregated eosinophils or Curschmann spirals composed of trapped epithelial cells. Incorrect Answers: Answer 1: Asbestosis is a cause of restrictive lung disease caused by reaction of the lung parenchyma to asbestos fibers. This disease can be seen in patients with occupational exposures such as renovating old houses or working in shipyards; however, the classic finding on histology is a golden fiber known as a ferruginous body. Answer 3: Chronic obstructive pulmonary disease can be seen in patients with an extensive smoking history such as seen in this patient; however, it tends to present in older patients and would not have eosinophilic crystals on histology. Answer 4: Lung cancer, especially the squamous cell and small cell varieties, are frequently seen in patients who have an extensive smoking history. Squamous cell lung cancer can be seen as keratin pearls and intercellular bridges. Small cell lung cancer can be seen as small, round, blue cells. Answer 5: Sarcoidosis is most commonly seen in African American individuals and can present with shortness of breath in middle age; however, this disease would be seen on histology with non-caseating granulomas. Chest radiograph will show large lymph nodes near the mediastinum. Sarcoidosis can also present systemically with heart block, lupus pernio, and joint pain.
A 32-year-old male presents to the emergency department because he coughed up copious amounts of bloody sputum at work. He also says that he has had multiple lung infections over the last 2 years. Furthermore, he says that during the episodes, he coughs up a lot of foul-smelling sputum. Past medical history is significant for childhood allergies and absence seizures. He has smoked 1 pack per day for 10 years and drinks socially. A computed tomography scan of this patient shows the section seen below. Which of the following diseases would most likely also cause the same symptoms as seen in this patient? A.Asbestosis B.Cystic fibrosis C.Emphysema D.Sarcoidosis E.Toxic fumes
B.Cystic fibrosis This patient with hemoptysis, recurrent infections, purulent sputum, and enlarged airways on computed tomography scan most likely has bronchiectasis, which can be caused by cystic fibrosis. Bronchiectasis is a form of obstructive lung disease where robust inflammatory responses lead to structural damage and mucous stasis within the lung. The stasis can further be colonized by bacteria generating chronic inflammatory conditions. Over time, the inflammation leads to permanent abnormal dilation of the bronchioles, which can be seen on both radiographs as well as computed tomography scans. Causes of bronchiectasis are varied but include causes of inflammation such as tuberculosis, tumor, ciliary dyskinesia, cystic fibrosis, and autoimmune conditions. Picture shows an axial computed tomography slice. The yellow circles show enlarged airways with thick bronchial walls characteristic of bronchiectasis. Incorrect Answers: Answer 1: Asbestosis can cause calcified plaques as well as restrictive lung disease; however, it will not cause bronchiectasis. Answer 3: Emphysema can cause obstructive lung disease; however, it will not cause bronchiectasis. Answer 4: Sarcoidosis can cause restrictive lung disease; however, it will not cause bronchiectasis. Answer 5: Toxic fumes can cause bronchiolitis obliterans; however, it will not cause bronchiectasis.
An 8-year-old male presents to the emergency department with an 8-hour history of fever and difficulty breathing. His past medical history is significant for recurrent pneumonia, shortness of breath, and diarrhea as an infant. His family recently immigrated to the United States and have not yet obtained their previous medical records. On presentation, his temperature is 101.6°F (38.7°C), blood pressure is 106/68 mmHg, pulse is 98/min, and respirations are 22/min. Physical exam reveals an ill-appearing boy with increased work of breathing and scattered rhonchi over both lungs fields. Sputum Gram stain and culture reveal a gram-negative, encapsulated bacteria that produces a blue-green pigment. Which of the following is most likely associated with the cause of this patient's disorder? A.Absence of thymic T lymphocyte development B.Decreased chloride membrane permeability C.Increased smooth muscle beta-2 adrenergic activity D.Inhibition of B lymphocyte maturation Reduced inhibition of metalloproteinase activity
B.Decreased chloride membrane permeability This patient who presents with fever and a blue-green pigment-producing organism most likely has Pseudomonas pneumonia. Recurrent infections with Pseudomonas species and diarrhea are consistent with cystic fibrosis, which is caused by decreased chloride membrane permeability.Cystic fibrosis is a genetic disorder caused by a mutation in the CFTR gene located on chromosome 7. CFTR is a transmembrane ion channel that is specific for chloride and is activated by cyclic adenosine monophosphate. In cystic fibrosis, the phenylalanine residue present at location 508 is deleted, resulting in protein instability and premature degradation within epithelial cells. The absence of CFTR on the cell surface subsequently leads to mucus dehydration and blockage of respiratory ducts as well as pancreatic ducts. These duct blockages lead to recurrent infections as well as exocrine dysfunction.Incorrect Answers:Answer 1: Absence of thymic T lymphocyte development is consistent with DiGeorge syndrome; however, this disorder would also present with abnormal facial features, cleft palate, hypocalcemia, absent thymus, and cardiac anomalies.Answer 3: Increased smooth muscle beta-2 adrenergic activity is consistent with asthma; however, asthma does not have an association with Pseudomonas nor would it present with recurrent diarrhea. Instead it presents with reversible bronchoconstriction that is associated with other atopic diseases such as eczema and allergies.Answer 4: Inhibition of B lymphocyte maturation is consistent with Bruton agammaglobulinemia, which may also present with recurrent infections by encapsulated bacteria; however, it would not present with rhonchi in all lung fields nor recurrent diarrhea. Instead, it presents with recurrent bacterial infections at 6 months of age when maternal antibodies decrease.Answer 5: Reduced inhibition of metalloproteinase activity is consistent with alpha-1-antitrypsin deficiency; however, this disease would present with liver injury and emphysema rather than recurrent lung infections.
A 21-year-old college student with a history of hypertrophic cardiomyopathy and a systolic murmur heard best at the left sternal border presents to the emergency room with a temperature of 102.3 F, worsening shortness of breath, chest pain and productive cough. He states that two days ago upon returning from Spring Break in Australia he drank a significant amount of alcohol and had an episode of vomiting. He recalls little about the evening. His electrocardiogram displays a regular sinus rhythm and his chest radiograph is shown. Which of the following is the most likely etiology of his illness? A.Long distance travel B.Impaired cough reflex C.Hypertrophic cardiomyopathy D.Valvular disease Close living quarters
B.Impaired cough reflex Aspiration pneumonia is associated with a diminished cough reflex and dense opacities in the dependent lung fields. Impaired cough reflex, as seen with alcohol ingestion, is a risk factor for aspiration pneumonia. Ingestion of alcohol or any substance that reduces consciousness can be associated with a decreased cough reflex and an inability to protect the airway, predisposing to aspiration pneumonia. Other conditions that can be associated with aspiration pneumonia include dementia, drug abuse, neuromuscular disorders and large volume feeding tubes. Chest radiograph displays an infiltrate in the lower right lung field (arrows) consistent with a dependent distribution due to aspiration pneumonia (next slide). Incorrect Answers: Answer 1: Long distance travel is often associated with the formation of pulmonary embolism. Answer 3: Hypertrophic cardiomyopathy is often asymptomatic and diagnosed incidentally by EKG. It can be a cause of sudden death. Answer 4: Valvular heart disease can be associated with IV drug use with the resulting formation of pulmonary embolism. Answer 5: Close living quarters such as army barracks or college dorm rooms are often associated with infectious diseases such as tuberculosis or meningococcemia.
A 45-year-old homeless male presents to the emergency room with a chronic cough. Past medical history is positive for alcohol abuse. A chest radiograph is taken and is shown. What is the pathologic mechanism for the characteristic appearance of this condition on imaging? A.Mast cell priming and degranulation B.Neutrophil activation and parenchymal destruction C.Cilia loss and dilatation of the bronchioles D.Granuloma formation and calcification E.Medium vessel vasculitis with bleeding into the airway
B.Neutrophil activation and parenchymal destruction The combination of chronic cough and a cavitary lung lesion with an air fluid level is characteristic of a lung abscess. This presentation is caused by neutrophil activation and resultant destruction of lung parenchyma and necrosis.Lung abscesses are most often the result of aspirated oropharyngeal material or bronchial obstruction. Therefore abscesses are most often seen in dependent areas of the lung and because of the wider and more vertical right mainstem bronchus, they are more commonly seen on the right than on the left. The most common pathogens include Bacteroides, Peptostreptococcus, and Fusobacterium. Given that this is a homeless man who is at high risk for TB, it should be noted that TB cavitary lesions are more frequently seen in the upper lobes of the lung. In general in certain patient populations (homeless, prison, immigrant) with pulmonary symptoms it is wise to screen a PPD for tuberculosis. Incorrect Answers: Answer 1: Mast cell priming and degranulation are involved in the pathogenesis of asthma. Answer 3: Cilia loss and dilatation of the bronchioles causes bronchiectasis. Answer 4: Granuloma formation and calcification are seen with sarcoidosis. Answer 5: Medium vessel vasculitis with bleeding into the airway is seen with granulomatosis with polyangiitis (Wegener's).
A 17-year-old male presents to the ED after a physical altercation that resulted in a stab wound. He was stabbed at level T12 in the back. On physical examination, there is significant muscle weakness affecting the whole right lower extremity. There is also impaired vibration and proprioception sense of the right leg and loss of pain and temperature sense of the left leg. What is the most likely diagnosis?
Brown-Sequard Syndrome
A 33-year-old male is seen in your emergency department complaining of cough and shortness of breath with recent fevers and chills. His CT scan is shown in Figure A. Which of the following risk factors most likely caused his disease? A.Occupational exposure B.IV drug use C.Alcoholism D.Smoking E.Poor diet
C.Alcoholism On CT, this patient has a cavitary lung lesion concerning for abscess. Alcoholism is a significant risk factor for lung abscesses, because it can predispose to aspiration of oral anaerobes such as fusobacterium, peptostreptococcus, and bacteroides.Poor dentition or a history of seizures may also predispose to aspiration pneumonia. Characteristically lung abscesses from aspiration occur in dependent portions of the right lung due to the shorter main-stem bronchus. Diagnosis of a lung abscess is made by chest x-ray and CT scan. Management includes 4-6 weeks of IV penicillin or clindamycin, drainage with percutaneous catheter insertion, or surgical removal with treatment failure. Incorrect answers:Answer 1: Occupational exposure to silica would result in multiple rounded upper lobe opacities.Answer 2: IV drug use may result in septic emboli that are seen on CT scan as multiple peripheral parenchymal nodules.Answer 4: Smoking may result in lung cancer, most commonly squamous cell cancers, which tend to be central masses that can produce lobar atelectasis.Answer 5: Poor dentition. not poor diet, would be expected to contribute to the pulmonary findings in this patient.
A 72-year-old man presents to his primary care physician with a 6-month history of shortness of breath. He says that he used to enjoy playing golf with his friends but now he cannot walk for long enough to play. The shortness of breath is now starting to complicate his ability to get around in daily life. His past medical history is significant for diabetes, hypertension, and early stage prostate cancer that was removed 10 years ago without recurrence. He has smoked 1 pack per day for the past 55 years and drinks about 6 drinks per week. On presentation he is found to be breathing with pursed lips. Physical exam reveals decreased breath sounds on auscultation, increased chest diameter, and hyperresonance to percussion. Which of the following findings would most likely be seen in this patient? A.Decreased residual volume and decreased 1 second forced expiratory volume B.Decreased residual volume and increased 1 second forced expiratory volume C.Increased residual volume and decreased 1 second forced expiratory volume D.Increased residual volume and increased 1 second forced expiratory volume E.Normal residual volume and decreased 1 second forced expiratory volume
C.Increased residual volume and decreased 1 second forced expiratory volume This patient with dyspnea, pursed lip breathing, decreased breath sounds, and hyperresonance to percussion most likely has emphysema, which would present with increased residual volume and decreased 1 second forced expiratory volume. Emphysema is a condition characterized by decreased elasticity and increased compliance of the respiratory tree resulting in dilation of air spaces. It is caused by destruction of the alveolar wall, which most commonly occurs after prolonged smoking. This destruction will result in collapse of airways during expiration so patients will often compensate by increasing airway pressure through pursed-lip breathing. Over time, chronic compensation will result in hyperinflated lungs and increased chest diameter. These compensatory changes will also lead to increased residual volume and decreased expiratory velocity. Incorrect Answers: Answer 1: Decreased residual volume and decreased 1 second forced expiratory volume is incorrect because emphysema leads to increased lung size. Restrictive lung diseases will often have decreased residual volume. Answer 2: Decreased residual volume and increased 1 second forced expiratory volume is incorrect because emphysema leads to increased lung size and collapse of airways leads to decreased expiratory velocity. Answer 4: Increased residual volume and increased 1 second forced expiratory volume is incorrect because collapse of airways in emphysema leads to decreased expiratory velocity. Answer 5: Normal residual volume and decreased 1 second forced expiratory volume is incorrect because emphysema leads to increased lung size. Lung size can be normal in some obstructive diseases such as transient bronchoconstriction.
A patient presents with difficultly breathing after adopting a new kitten. The cell pictured in picture below plays a role in what phase of this disease process? A.Sensitization B.Early Activation C.Late Activation D.Resolution Remodeling
C.Late Activation The vignette above most likely describes extrinsic asthma, due to a response generated by pet dander. Eosinophils play a key role during the late activation phase.Type I hypersensitivity asthma results from a physiologic response to an inhaled (external) allergen which proceeds by specific stages: sensitization, early activation, and late activation. Late activation results in the activation of eosinophils which were recruited by eotaxin released during early activation. Incorrect Answers:Answer 1: Sensitization is mediated by dendritic cells and Th2 CD4+ cells.Answer 2: Early activation is mediated by mast cells.Answer 4: Resolution is mediated by macrophages.Answer 5: Remodeling is mediated by epithelial cells Picture shows an eosinophil.
A 40-year-old man with persistent moderate asthma presents for a pulmonary function test. His ratio of forced expiratory volume in one second (FEV1) to forced vital capacity (FVC) is 0.69, and his FEV1 is 65% of his predicted values. What other findings can be expected in the remainder of his pulmonary function test? A.Decreased diffusion limitation of carbon monoxide (DLCO) B.Increase in FEV1 with methacholine C.Decrease in FEV1 with albuterol D.Increase in fractional exhalation of nitric oxide E.Decrease in total lung capacity
D.Increase in fractional exhalation of nitric oxide This vignette assesses knowledge of asthma pathophysiology as it manifests on pulmonary function testing. In asthmatic patients, fractional exhalation of nitric oxide increases. Asthma is defined as a reversible airflow obstruction process due to inflammation. Assessment of nitric oxide hinges on inflammation. Nitric oxide is produced by endothelial cells in response to bronchoconstriction and inflammation. High levels of exhaled nitric oxide (at least 50 parts per billion) suggest airway inflammation, suggestion potential asthma. Incorrect Answers: Answer 1: DLCO is often normal to elevated in a patient with asthma. Answer 2: FEV1 should decrease with methacholine, a cholinergic (and thus bronchoconstrictor). Answer 3: FEV1 should increase with albuterol, a beta2 agonist (and thus bronchodilator). Answer 5: Total lung capacity is normal to mildly elevated in a patient with asthma.
A 35-year-old female presents to her primary care provider complaining of fever and cough with symptoms starting five days ago. She returned from a cruise to the Bahamas two weeks ago. She initially experienced fever and dyspnea with a productive cough with yellow-green sputum and noticed blood in her sputum one day ago which prompted her to visit the physician. The patient drinks 2-3 alcoholic beverages per day and has a 10 pack-year smoking history. Her temperature is 101°F (38.3°C), blood pressure is 130/70 mmHg, pulse is 100/min, and respirations are 23/min. On examination, the patient is diaphoretic with notable increased work of breathing. Rales are noted in the right middle lung zone. A chest radiograph is shown. A biopsy of the lesion would likely reveal which of the following? A.Type II pneumocyte hyperplasia with interstitial fibroblastic proliferation B.Intra-alveolar intact erythrocytes with neutrophils and fibrin C.Dense fibrosis with architectural destruction D.Intra-alveolar fragmented erythrocytes with neutrophils and fibrin E.Vascular engorgement, intra-alveolar fluid, and numerous bacteria
D.Intra-alveolar fragmented erythrocytes with neutrophils and fibrin The patient in this vignette presents with fever, dyspnea, a productive cough, and focal lung consolidation suggestive of lobar pneumonia. After 5 days of lobar pneumonia, the lung is likely undergoing grey hepatization, characterized on histology by fragmented erythrocytes with an alveolar exudate containing neutrophils and fibrin.Classic patterns of pneumonia include lobar pneumonia, bronchopneumonia, and interstitial pneumonia. Lobar pneumonia follows a common progression over time as the intra-alveolar exudate consolidates and eventually resolves. Following congestion (first 24 hours) and red hepatization (2-3 days), gray hepatization (4-6 days) occurs. In this latter stage, the affected lobe(s) may appear macroscopically as gray-brown and firm. Microscopically, the alveoli may be filled by fragmented erythrocytes, neutrophils, and fibrin. Resolution is the final stage after gray hepatization.Picture demonstrates the characteristic appearance of lobar pneumonia. Note the focal consolidation in the right middle to lower lung zones near the heart border. The opacity demonstrates a straight upper border suggesting lobar involvement with clear separation along the horizontal fissure. This is suggestive of a right middle lobe pneumonia which would be confirmed with a lateral radiograph. Incorrect Answers:Answer 1: Interstitial pneumonia is a form of pneumonia that is distinct from lobar pneumonia. This condition is characterized histologically by type II pneumocyte hyperplasia with interstitial fibroblastic proliferation.Answer 2: The second stage of lobar pneumonia is red hepatization (2-3 days). This occurs when erythrocytes extravasate into alveolar spaces along with neutrophils and fibrin. Macroscopically, this makes the lung appear red and firm (liver-like).Answer 3: Interstitial lung disease is characterized by dense fibrosis that often destroys normal lung architecture. This is a chronic disease and is inconsistent with the acute process described in this vignette.Answer 5: The first stage of lobar pneumonia is congestion (first 24 hours). This is characterized microscopically by vascular dilatation with an alveolar exudate containing mostly bacteria.
A tall, 25-year-old male is brought to the ED by his friend after sudden difficulty breathing while smoking a cigarette. In the trauma bay he is tachypneic, but able to talk to you. Vital signs show that he is afebrile and tachycardic with blood pressure of 115/60. Physical exam reveals hyperresonance and absent breath sounds over the left upper lung. A chest x-ray is obtained and shown below. What is the most likely diagnosis? A.Tension pneumothorax B.Left lower lobe pneumonia C.Left upper lobe cavitation D.Spontaneous pneumothorax E. Nondiagnostic, further imaging required
D.Spontaneous pneumothorax This vignette describes a young man with acute dyspnea in the absence of trauma. He has a spontaneous pneumothorax on chest x-ray.A pneumothorax presents with hyperresonance and absent breath sounds unilaterally. If it is not caused by trauma, it is likely a primary spontaneous pneumothorax. It is often seen in tall thin young men and associated with rupture of apical subpleural blebs. Smoking is a risk factor for spontaneous pneumothorax. Physical exam may further reveal subcutaneous emphysemas. Management usually involves observation until symptomatic resolution, but large pneumothorax reduction can be expedited with supplemental oxygen.Incorrect Answers:Answer 1: Tension pneumothorax manifests often in the presence of trauma, causing mediastinal deviation and cardiovascular collapse.Answer 2: The left lower lobe appears more consolidated due to the presence of apical air. However, this patient has no symptoms or signs suggestive of pneumonia.Answer 3: The left upper lobe appears hyperlucent because of air. Cavitation should be accompanied by more distinct tissue silhouette margins.Answer 5: This chest x-ray is sufficient to diagnose pneumothorax. Further imaging is not indicated.
A 15-year-old boy is brought to the emergency department with a 30 minute history of difficulty breathing. He was playing basketball in gym class when he suddenly felt pain in the right side of his chest that got worse when he tried to take a deep breath. Physical exam reveals a tall, thin boy taking rapid shallow breaths. There are decreased breath sounds in the right lung fields and the right chest is hyperresonant to percussion. Which of the following is true of the lesions that would most likely be seen in this patient's lungs? A.Related to liver failure B.Related to smoking C.Found in the lower lobes D.Found in the upper lobes E. Found near the pleura
E. Found near the pleura This tall, thin, young man with unilateral pleuritic chest pain and hyperresonance most likely experienced spontaneous pneumothorax. This is associated with paraseptal emphysema with bullae near the pleura.Emphysema is a group of conditions that feature dilation of air spaces due to weakening of wall elastin. This results in decreased elasticity and increased compliance of the airway structures. Emphysema can be centriacinar with dilated respiratory bronchioles, panacinar with dilated alveoli, or paraseptal with bullae near the pleura. Paraseptal emphysema most commonly involves young, otherwise healthy males and does not obstruct the airway. Therefore, these patients will usually present only when 1 of these bullae rupture and lead to pneumothorax.Incorrect Answers:Answer 1: Related to liver failure is characteristic of panacinar emphysema, which is often caused by alpha-1-antitrypsin deficiency.Answer 2: Related to smoking is characteristic of centriacinar emphysema, which is typically seen in older smokers.Answer 3: Found in the lower lobes is characteristic of panacinar emphysema because this portion of the lung has the highest perfusion and would sustain the most damage in alpha-1-antitrypsin deficient patients.Answer 4: Found in the upper lobes is characteristic of centriacinar emphysema because these lobes often get high exposure to smoke in smokers.
A 33-year-old female complains of shortness of breath on exertion, which has progressed over the last year. Review of systems is positive for fatigue. She has not traveled recently but immigrated from Africa 6 years ago. Family history is notable for asthma in her father (diagnosed at age 10) and heart failure in her mother (diagnosed at age 39). Her temperature is 99°F (37°C), blood pressure is 130/80 mmHg, pulse is 80/min, and respirations are 20/min. Chest radiograph is shown below. Which pathological process best explains the findings seen in this patient? A.Bacterial proliferation within alveolar macrophages B.Fibroblast proliferation and collagen deposition within alveolar walls C.Plaque formation within coronary vascular intima D.Smooth muscle contraction within bronchioles E.Smooth muscle proliferation within vascular media
E.Smooth muscle proliferation within vascular media This young/middle-aged woman with exertional dyspnea and fatigue, family history of premature heart failure, and pulmonary artery dilation on chest radiograph, likely has pulmonary arterial hypertension (PAH), which involves vascular smooth muscle proliferation within the pulmonary arteries.Pulmonary hypertension is defined as pulmonary arterial pressure ≥ 25 mmHg. While many pulmonary and cardiac diseases (e.g., left heart failure) can secondarily elevate pulmonary vascular pressure, pulmonary arterial hypertension (PAH, formerly called primary pulmonary hypertension) is a proliferative vasculopathy of the pulmonary arterioles. In PAH, smooth muscle hyperplasia and hypertrophy increase pulmonary arteriole resistance and pressure, which further damages the arteriole walls in a repetitive cycle. PAH may be acquired, such as through toxin/drug or infectious damage to pulmonary arterial walls (e.g., Schistosoma infection), or inherited via mutations in anti-proliferative or apoptotic proteins. PAH can cause pulmonary artery dilation and right ventricular hypertrophy, both of which may be seen on chest radiography. Eventually, PAH can cause right heart failure. Pulmonary artery pressure can be estimated with an echocardiogram, and the diagnosis confirmed with right heart catheterization. Incorrect Answers:Answer 1: Bacterial proliferation within alveolar macrophages occurs in pulmonary tuberculosis, which typically presents with cough, fever, and chest radiograph showing cavitary lesions.Answer 2: Fibroblast proliferation and collagen deposition within alveolar walls describe pulmonary fibrosis. It too presents with dyspnea on exertion, but chest radiograph shows diffuse (usually reticular or "honeycomb") pulmonary interstitial abnormalities.Answer 3: Plaque formation within vascular intima is part of atherosclerosis. While atherosclerosis of the pulmonary arteries can contribute to pulmonary hypertension, atherosclerosis of the coronary arteries alone would not; rather, it could cause chronic angina or myocardial infarction.Answer 4: Smooth muscle contraction within bronchioles (i.e., bronchoconstriction) occurs with asthma, causing intermittent dyspnea, wheezing, and/or coughing in response to triggers (e.g., exercise and common allergens). Given the absence of wheezing and presence of pulmonary artery dilation, asthma is less likely here than PAH.
A 68-year-old male visits the medical office complaining of trembling hands and difficulty in walking. The patient is married and lives at home with his wife of 40 years. Their three children have left home but still live in the same town. The patient had kidney stones 12 years ago but has not had any other significant health problems. Neurologic examination: Resting tremor of the left hand that diminishes when performing a task. During passive movement of the left arm, the muscles are rigid, causing "cogwheel" motion during stretching. Facial expressions are reduced. When walking, the patient has difficulty taking the first step but then is able to walk smoothly with a shuffling gait. What is the most likely diagnosis?
Parkinson's
An 82-year-old man is brought to the emergency department by ambulance with a 2-hour history of increased difficulty breathing at home. His wife says that over the last day he started having more phlegm production and shortness of breath. He started gasping for air this morning at which point his family became concerned and called 911. Chart review reveals a previous myocardial infarction 4 years ago and social history reveals a 30 pack-year history of smoking as well as a 40-year history of working at a shipyard. Physical exam reveals decreased breath sounds with faint bilateral wheezing and non-distended neck veins at 45 degrees upper body elevation. Despite treatment, the patient passes away. Which of the following would most likely be seen in post-mortem examination of this patient's lungs? A.Alveolar macrophages containing hemosiderin deposits B.Alveolar septum containing ferruginous bodies C.Centrilobular airspace dilation and mucous gland proliferation D.Panacinar dilation of air spaces and destruction of parenchyma Small round blue cells sheets with limited organization
This patient who presents with dyspnea, bilateral wheezing, and a 30 pack-year history of smoking most likely has an exacerbation of chronic obstructive pulmonary disease. This could be seen on histology as centrilobular airspace dilation and proliferation of mucous glands. Chronic obstructive pulmonary disease (COPD) is a chronic condition characterized by dilation of air spaces as well as proliferation of mucous glands. These changes are caused by chronic inflammation of airways and subsequent destruction of alveolar walls. COPD is strongly associated with a history of smoking, and patients may present with a predominant phenotype of emphysema or bronchitis. Emphysematous patients ("pink puffers") are acyanotic with pursed-lip breathing and loss of lung markings on radiograph. Chronic bronchitis patients ("blue bloaters") present with cyanosis, productive cough, and edema. Patients with COPD can have acute exacerbations triggered by infections and should be treated with albuterol/ipratropium nebulizers, magnesium, corticosteroids, and antibiotics (if associated with pneumonia) during acute flares.
A 37-year-old African American female complains of a persistent cough, fever, eye-pain and arthralgias. Physical examination is notable for skin findings shown. A chest radiograph is performed. Which of the following would most likely be found in this patient? A.Increased serum parathyroid hormone B.Decreased serum angiotensin converting enzyme level C.Caseating granulomas on nodule biopsy D.Increased CD4:CD8 in bronchioalveolar lavage fluid E.Ipsilateral facial muscle contraction after tapping anterior to the external auditory meatus
This patient's clinical presentation, along with dermatologic and chest radiograph findings are highly suggestive of sarcoidosis. Of the choices provided, an increased CD4:CD8 in bronchioalveolar lavage (BAL) fluid is likely to be found. Sarcoidosis is a noncaseating granulomatous disease that affects many organ systems. When it affects the lung, it typically causes restrictive lung disease. Patients with sarcoidosis can be completely asymptomatic, or have organ involvement. On chest imaging, bilateral hilar adenopathy is a classic feature. The etiology of sarcoidosis is unclear; however, it seems that genetically predisposed people may have a dysregulated immunologic response to an unknown antigen. T-cells play an important role in the development of sarcoidosis. CD4+ T-cells interact with antigen-presenting cells, eventually leading to IL-2 and IFN-y release, causing T-cells to expand, and macrophages to activate. Granuloma formation subsequently develops. Incorrect Answers: Answer 1: An increased parathyroid hormone (PTH) would not be expected in sarcoidosis. Hypercalcemia is secondary to increased 1-alpha-hydroxylase activity in macrophages, which increases activated Vitamin D levels. Hypercalcemia would suppress PTH release directly. Answer 2: Decreased serum angiotensin converting enzyme (ACE) level is incorrect. Though ACE levels are non-specific in sarcoidosis, about 75% of untreated patients would have their ACE levels elevated. Answer 3: Caseating granulomas would be seen in diseases such as tuberculosis. Noncaseating granulomas is classic for sarcoidosis. Answer 5: Ipsilateral facial muscle contraction post tapping the facial nerve describes Chvostek sign, a sign of hypocalcemia. One would expect hypercalcemia in sarcoidosis due to increased activated vitamin D levels.
A 35-year-old female complaining of difficulty in walking. She indicates she had trouble walking for 5 days last month and that she has experienced pain and prickly sensations that come and go, as well as occasional muscle weakness. The patient had difficulty in balancing and maintaining a normal gait. Hypotonic muscles. Normal patellar tendon reflexes. MRI: Gadolinium-enhancing brain lesions, one lesion in infratentorium and three periventricular lesions. Cerebrospinal fluid analysis: Oligoclonal IgG band Edrophonium (Tensilon) test: Normal Nerve conduction velocity test: Normal Electromyography: Normal What is the most likely diagnosis?
myasthenia gravis